LSAT Question Analysis #5

It’s an ugly, rainy, windy day in Washington and all of my lessons are canceled. I’m going to dig into 10 New Actual, Official LSAT PrepTests and do a logic game to pass the time. Let’s try PrepTest 57, Section 1, Questions 1-5 (10 New LSAT, page 188).

 

This looks like a relatively straightforward ordering game. The items are G, H, J, K, L, and M. I’ll shorthand the rules as follows:

  • (1) HG
  • (2) K-G
  • (3) M-L
  • (4) JM or MJ

 

The repetitions of G between rules 1 and 2 and M between rules 3 and 4 are surely not accidental. We can combine rules in each case to obtain the following partial orderings:

  • (1)+(2) K-HG
  • (3)+(4) JM-L or MJ-L

 

Let’s not dally with the rules any further. Question 1 is the usual complete and accurate list. The strategy for this question type never changes. Go through the rules one-by-one and eliminate answers that fail the rules in turn. Don’t forget to mark the rule number used in each case, to aid in checking your work should anything go awry.

Rule 1 eliminates answer E, since the item preceding G is K, instead of H. Rule 2 eliminates answer B, since K is last and does not precede G. Rule 3 eliminates C, since M comes after L. Finally, rule 4 eliminates A, since M is surrounded by G and L and not by J. So D is correct.

 

Question 2 doesn’t add any information to the question, merely asking about the third item. The lack of additional information generally makes these questions tougher, since we don’t have anything to grab onto to know where to begin. Our best bet is to start with the most-restricted item(s), and since third is closer to the beginning than to the end, we should probably start with something like G or L (each of which must have two items preceding it).

However, the list K H G J M L places G third, and the list J M L K H G places L third. These orderings weren’t too difficult to produce, since the combined rules 1+2 share no items with, and are completely independent of, the combined rules 3+4. With G and L considered and rejected, we’re adrift again.

But notice the two blocks, HG and JM/MJ. Of the two, the HG block is the least flexible, because it can’t switch orders like the JM/MJ block can. So H is also fairly restricted. If it were third, G would have to be fourth: _ _ H G _ _.

Now where would be put the JM/MJ block? It can’t go first and second, because K has to fill one of those spots. But it also can’t go fifth and sixth, because L has to fill one of those spots. So we’re stuck; H must not be able to be third. And that’s the answer we’re looking for: B.

 

Question 3 continues this game’s annoying pattern of not providing additional information (probably because the rules ultimately comprise two completely independent partial orderings, making any question with additional information rather simplistic).

In a “must be” question like this one or the one before it, we’re looking to use the rules to determine the right answer. Process of elimination would be a bad idea. But we have no guidance about where to begin. So let’s skim through the answer choices. Answer A gives H fourth, which places G fifth. This isn’t obviously a problem, so let’s keep moving. Answer B given J fourth, which places M either third or fifth. This offers even less restriction, so again we’ll move on. Note that we’re not ruling these answers out. We’re simply setting them aside as weak candidates — but we won’t waste the effort to prove them wrong, and we would come back to them later if necessary.

Answer C places K second. This one matters. There are two blocks to place (games like this are all about the blocks!) — HG and JM/MJ. But if K is second, then those two blocks must go third/fourth and fifth/sixth in some order: _ K block block. The only remaining item is L, and the only remaining slot is first, but L cannot be first since it has to follow the JM/MJ block. So this ordering is impossible, K cannot be second, and the answer is C.

As a followup, a quick glance reveals no obvious reason why L couldn’t be third (recall that we placed L third successfully in question 2) or M couldn’t be second (again, we actually did this successfully in question 2).

 

Question 4 once again offers no additional information. We’re most likely playing with blocks again. Fifth is pretty late, so we might immediately think of items that must come earlier in their lists. K, for instance, obviously cannot be fifth, since two items follow it. Sadly, K is the one item not present in any answer choice. We could certainly check H, J, and M, since each of those items must be followed by at least one other, but we could again be disappointed. Instead, think back to the outcome of question 3. LSAT logic games frequently test students’ ability to learn and to apply lessons from one question to the next. What did we learn in question 3? Well, we found that placing an item second could force the two blocks into slots three through six, causing problems for the remaining item. Now we’re asked about the fifth slot — an exact mirror image of the second. Surely this cannot be a coincidence…?

Imagine the list with a mystery item fifth: _ _ _ _ X _. Following the logic of question 3, if mystery item X isn’t a member of either block, then the two blocks HG and JM/MJ must appear first through fourth in some order: block block X _. One item would remain, and it would have to go last. Well, what item isn’t in a block and can’t go last? K. And if we want to force the blocks to appear first through fourth and leave K to be stuck in sixth, then the mystery item must be the only other item available, and that item is L. To sum it all up, L cannot be fifth, because if it were then the blocks would have to take first through fourth and K would be stuck with only the unacceptable sixth slot left. Thus, the correct answer is D.

 

Question 5 wraps up the game by asking what other rule could replace rule 3 without changing the conditions of the game. Rule 3 states M-L, and we combined it with rule 4 (JM or MJ) to get JM-L or MJ-L. If we need another rule to push M before L, then it must have something to do with the fact that M and J form a block. Perhaps you already have an idea of what we’re looking for. Perhaps you don’t. Either way, let’s turn to the answer choices for help.

Answer A, L near the end (last three), doesn’t do much for us at all. It could still come before the JM/MJ block. Furthermore, this new rule would eliminate certain possibilities that were allowed before. Remember that we managed to place L third in question 2.

Answer B, L next to J, is inconsistent with the combined rule that allowed JM-L.

Answer C, J before L, combined with M next to J, means that JM/MJ must come before L. This is the same combined rule that we derived from the original rules 3 and 4, making this answer choice, C, the one we need.

Briefly, glance at answers D and E. D is out of left field, tying L to H — something entirely absent from the original rules of the game. E is in direct conflict with the original combined rule JM-L or MJ-L. So C is confirmed as the correct answer.

 

Tags: , , , ,

Comments

Your email address will not be published. Required fields are marked *